Help! I can't wrap my head around this one AT ALL. I'm not even sure what the conclusion is.
http://classic.7sage.com/lsat_explanations/lsat-60-section-3-question-24/
174 posts in the last 30 days
Help! I can't wrap my head around this one AT ALL. I'm not even sure what the conclusion is.
http://classic.7sage.com/lsat_explanations/lsat-60-section-3-question-24/
I have been doing a good amount of practicing with logic games this past week or so and I been noticing that grouping games with more than two groups have been giving me a lot of trouble. Is there any way to approach them without having to resort to creating a sub-diagram for each question?
For the logic games where the question asks you to replace a rule with another rule to maintain the same effect on the game...
I know that you can't replace it with a new rule that allows for a possibility that the old rules would not allow (wouldn't be the same effect). But, is it also true that you can't have a new rule that is a subset of the old rules? As in, the old rules allow for more possibilities than the new rule. That wouldn't be the same effect either, right? So you would have to find the same set of possibilities?
If that makes any sense..
When I am taking my practice tests I often come across questions (in LR sections, specifically) where I am certain that 3 of the answers are wrong, but am unable to clearly distinguish between the other two. For time's sake I usually put the answer that seems best and move on. Somehow, at a percentage that seems mathematically impossible for a 50/50 scenario, I miss almost every one of these questions. Is there any tricks, or specific things to look for in order determine the better of two decent answers?
Over the past year I know that some people have mentioned seeing grouping games with elements reused. I know this isn't a recent trend (many of the older games have multiple groups 3+ with each group getting at least one or up to 3 items).
However, the game type I am referring to is your standard in out game (2 groups only) but just because one element is in the "in" column, it doesn't necessarily mean it can't be in the "out" column. Nowhere in the scenario does it say that "each element will be used exactly once." Basically, the elements can be "reused." Also, the number of elements aren't limited to just 3, but range from 5-7.
Some ppl have told me that the test put a twist by splitting the elements into 2 subgroups (i.e., women vs. men)... and adding in a weird rule (i.e., chairperson in group 1 can't be in group 2)...
What is the best way to prepare for this type of game? I recall seeing this type of game just once (PT25-S3-G1), but its a very straight fwd grouping game with a twist in which there must be at least one member shared...
Wow, I did everything on this question and got sucked into answer choice E but had no idea to interpret the word "that" the way JY explained it. Are there others words that indicate "and" or sufficiency that are worth knowing?
http://classic.7sage.com/lsat_explanations/lsat-62-section-2-question-19/
Anybody knows any tricks or tips to ace the games section?
How I read the stimulus was: say we let Q be the statement "create virtuous people"
Glen: I believe Q is most important because P (an alternative) is not desirable
Sara: But Q is more dangerous than P
I can't seem to get past why answer choice C is wrong. Glen's closing sentence states he endorses law's primary role to create virtuous citizens implying there is negligible danger in making Q the most important. On the other hand Sara counters by saying this is more dangerous than being overprotective of individual rights, thereby implying she believes there is an inherent danger in the government deciding what constitutes virtuous behavior. Wouldn't this point be something they directly disagree about?
So E summarizes Glen's argument, but Sara simply disagrees by stating Q is more dangerous than an alternative and seems to imply Q may not be the most important. But I feel that the level of interpolation to go from Sara's argument (Q being more dangerous) to Sara believing Q is not the most important is the same as that for C.
http://classic.7sage.com/lsat_explanations/lsat-56-section-2-question-17/
An Eagle's Eye for Detail is needed for this one.
I understood the question.
Study people easily angered--> more likely to have permanent high blood pressure--> More likely to get Heart Disease
Conclusion: Heart Disease can result from psychological factors (Anger)
I see E and look at it, and think it says "Psychological factors" cause both anger and high blood pressure. I then, think to myself, well, that wouldn't weaken the argument because that'd essentially cement the conclusion that "Psychological factors" caused heart disease.
Then, I looked at E again, it said "PHYSiological" which are different from "PSYchological" factors.
To summarize, have an eagle's eye for detail.
http://classic.7sage.com/lsat_explanations/lsat-55-section-1-question-22/
Hi, general question about strategy here. I can manage to solve almost every game in practice, but my problem is doing it fast enough to complete an entire section.
I think I have the understanding down; I suspect the problem is being too thorough (ex.double checking each answer on a CBT question, drawing all the possibilities when I don't need to).
Does anybody have tips/habits on how to go faster?
Would it be possible to get some advice on this question? I'm lacking a way to do this problem methodically and quickly.
http://classic.7sage.com/lsat_explanations/lsat-40-section-3-question-26/
Not quite sure if this is the appropriate category to ask this question, but here goes...
I just finished the "Intro to Arguments" lesson and wanted more practice, so I returned to the June 2007 Practice Test. As I went through Section 2, I came a cross a few questions where I wasn't sure how to parse out the Premise (P), Conclusion (C), or Context. As you can probably tell, I'm new at this, so if any of you find it easy to explain one problem or another, any help offered will be much appreciated. :)
#5. --- (My problem = P? C? Is this an argument?)
#11. -- (My problem = is the first sentence "Context" or part of the "Argument (P)?")
#18. -- (My problem = what is P and C here?)
#20. -- (My problem = where does Context end? with "citywide opposition." or with "opposing the new water system,?")
#22. -- (My Problem = where does the C start? with "This has the effect..." or with "which in turn discourages...?")
#24. -- (My Problem = Is C missing?)
I tried to diagram the premise in this question and I get R-->BOC. I picked an Idea "not rational" and negated it and made it the sufficient. But this is quite different from what you give in the video explanation (not BOC -->R) . What am I doing wrong? HELP
http://classic.7sage.com/lsat_explanations/lsat-64-section-3-question-26/
Hello, I was hoping you could help me out with this question. I originally had it down to C and E and chose E for the following reason:
Wouldn't answer choice E strengthen the conclusion in that companies that obtain would be more likely to perform the actions the economist says they will? More specifically, should E not be true, then would the argument possibly fall apart? The stimulus claims that companies CAN do this and that, which in turn benefits the consumer. Yes the companies can perform those actions but what if they don't? Doesn't answer choice E bridge the gap and make it more likely that companies will actually perform the actions they have the ability to do due to the monopoly?
http://classic.7sage.com/lsat_explanations/lsat-67-section-4-question-04/
On could be true questions, I can't find the exact question, but I remember it having an answer choice that provided a Must be true answer, and one that provided a could be true answer.
The must be true was incorrect, the could be true was correct.
However, other could be true questions I've experienced had a correct answer that was Must be true.
Would someone please help me on this?
The only way for me to make sense of it is to assume that when both a must be true and also a could be true answer are provided, select the could be true. In cases where there is only a must be true, select it.
I'm confused because I've read that could be true covers 1-100% which would include must be true answers, yet it was incorrect provided that a less certain answer choice was present.
Two weeks til the test.. anyone got a list of 5-10 Logic Games that are unordinary or very difficult? Want to make sure I don't get thrown off on Test day.
Thanks!
Hi All,
I was wondering if anyone had on advice of how to best practice logical reasoning with a mix of questions? I'm fine on accuracy and timing if I focus on a particular question type like on the drills, but I'm loosing accuracy once I take the timed practice tests. Is there a good method anyone has figured out to practice the mixed questions that minimizes taking out sections from the practice tests? (I don't want to use too many sections LR of the practice tests, since I want to save them for use as practice tests).
Thanks,
Michelle
Could someone please help me to find where in the course I can find how to deal with "logically complete question stem". I am working on PT 59 S2 Q24 and I think I misinterpreted what was required. Are these questions MBT or MC?
Pure sequencing game. Rule was something like W is before S OR S if before T BUT NOT BOTH... I was drawing a blank for a while. Doesn't that mean, in a nutshell, S will never be b/w W & T either before both or after both? Also, how do you draw that into your grand scheme efficiently? This game should have taken 6m, but drained 12 due to me not handling those 2 "or/not both" rules well.
Ok..I feel there is just no way I could have got this question. It is a most strengthen question and I feel the answer choice weakens the argument. The job is to sure up the fact that a volcanic event is the probable cause rather than meteorites. The part of B that says 'no known natural cause would likely account ...' I was thinking isn't meteorite as well as volcanoes natural causes so I immediately eliminated it. What do you guys think about this question?
http://classic.7sage.com/lsat_explanations/lsat-58-section-4-question-23/
Hi, I come across this problem in argument part questions: is a hypothesis the same thing as a statement?
I usually eliminate the wrong answer choices by matching them up to the stimulus (Ex. I eliminate the answer choice starts with "it is a premise " because I id'ed the statement as a conclusion). But some argument part questions describes statements differently (A) "This is a statement" or B) "this is a hypothesis for which the author supports." Should we eliminate answer choices that call it a hypothesis when it is a statement? Is there even a difference.
Ex: Is a "Fetuses develop fingernails in the third week of development" a statement?
Is " Scientists say fetuses develop fingernails in the third week" a hypothesis?
This question is very difficult because I just cannnot understand the stimulus. Could I please get an explanation of this question stem and answer choices?
First sentence I diagrammed as: wind & below 84-> pleasant. The second part of this question I diagrammed as : above 84 or no wind ->oppressive. I incorrectly got answer choice (B) it has something to do the the humidity but I was able to add it to my diagramming. Help please.
I do not understand the difference between answer choice C and D. How do you suggest that I decipher between two choices that are so close. I tried to incorporate them into the passage and incorrectly thought that D was a winner.
I've noticed that in some LR questions, there would be sentences that start off like "Even though","although". These sentences are qualifying the the argument a bit more correct? I remember in one particular video something about this, I think it was acknowledging something so as to prevent the reader from using it against the argument or something like that...
What are your opinions on this?